1
$\begingroup$

Some one know something about the prime numbers of the form $p = 2^k + 2^n +1$, I would like to know if is it possible give conditions to $n$ and $k$ for $p$ will be a prime.

$\endgroup$
3
  • 1
    $\begingroup$ oeis.org/A081091 $\endgroup$
    – Charles
    Jun 10, 2012 at 6:38
  • $\begingroup$ At least one of k and n should be odd. You might find it of interest to rule out divisibility by small primes. Gerhard "Ask Me About System Design" Paseman, 2012.06.09 $\endgroup$ Jun 10, 2012 at 6:49
  • $\begingroup$ Too vague and no motivation, vote to close. $\endgroup$
    – user9072
    Jun 10, 2012 at 8:02

1 Answer 1

2
$\begingroup$

Hello,

For each odd prime $q$, there should be conditions on $n,k$ mod (the order of $2$ mod $q$) that are necessary conditions for $q$ to not divide $2^n+2^k+1$. For $q=3$, the idea is implied in @Charles' comment on @joro's answer to this question.

For example, for $q=3$, $n,k$ should not both be even. For $q=5$, $n,k$ should not both be $1 \bmod 4$, and it should not be the case that one of $n,k$ is $3\bmod 4$ while the other is $0 \bmod 4$. For $q=7$, it should not be the case that one of $n,k$ is $1 \bmod 3$ while the other is $2 \bmod 3$. For $q=11$, it should not be the case that one of $n,k$ is $1 \bmod 10$ while the other is $3 \bmod 10$, and it should not be the case that one of $n,k$ is $6 \bmod 10$ while the other is $0 \bmod 10$, and it should not be the case that one of $n,k$ is $7 \bmod 10$ while the other is $8 \bmod 10$, and it should not be the case that one of $n,k$ is $2 \bmod 10$ while the other is $9 \bmod 10$, and it should not be the case that $n,k$ are both $4 \bmod 10$.

In general, if Artin's conjecture on primitive roots is true, there are infinitely many necessary conditions; one gets necessary conditions for each such $q$ where $2^{(q-1)/2} \equiv -1 \bmod q$, because this translates to $q-2$ values that $(n,k)$ cannot take in $(\mathbb{Z}/(q-1)\mathbb{Z})^2$, one of which is the condition that $n,k$ should not both be $(q-3)/2 \bmod q-1$.

$\endgroup$

Your Answer

By clicking “Post Your Answer”, you agree to our terms of service and acknowledge you have read our privacy policy.

Not the answer you're looking for? Browse other questions tagged or ask your own question.